Những câu hỏi liên quan
Dung Đặng Phương
Xem chi tiết
Phùng Minh Quân
25 tháng 1 2020 lúc 21:05

1) \(\Sigma\frac{a}{b^3+ab}=\Sigma\left(\frac{1}{b}-\frac{b}{a+b^2}\right)\ge\Sigma\frac{1}{a}-\Sigma\frac{1}{2\sqrt{a}}=\Sigma\left(\frac{1}{a}-\frac{2}{\sqrt{a}}+1\right)+\Sigma\frac{3}{2\sqrt{a}}-3\)

\(\ge\Sigma\left(\frac{1}{\sqrt{a}}-1\right)^2+\frac{27}{2\left(\sqrt{a}+\sqrt{b}+\sqrt{c}\right)}-3\ge\frac{27}{2\sqrt{3\left(a+b+c\right)}}-3=\frac{3}{2}\)

Khách vãng lai đã xóa
Nyatmax
25 tháng 1 2020 lúc 22:23

2.

Vỉ \(ab+bc+ca+abc=4\)thi luon ton tai \(a=\frac{2x}{y+z};b=\frac{2y}{z+x};c=\frac{2z}{x+y}\)

\(\Rightarrow VT=2\Sigma_{cyc}\sqrt{\frac{ab}{\left(b+c\right)\left(c+a\right)}}\le2\Sigma_{cyc}\frac{\frac{b}{b+c}+\frac{a}{c+a}}{2}=3\)

Khách vãng lai đã xóa
Nyatmax
26 tháng 1 2020 lúc 8:21

Cho o dong 2 la x,y,z nhe,ghi nham

Khách vãng lai đã xóa
Prissy
Xem chi tiết
zZz Cool Kid_new zZz
15 tháng 9 2020 lúc 17:30

Ta dễ có:\(\frac{1}{a^2+1}=\frac{a^2+1-a^2}{a^2+1}=1-\frac{a^2}{a^2+1}\ge1-\frac{a^2}{2a}=1-\frac{a}{2}\)

Một cách tương tự \(\frac{1}{b^2+1}\ge1-\frac{b}{2};\frac{1}{c^2+1}\ge1-\frac{c}{2}\)

Khi đó: \(\frac{1}{a^2+1}+\frac{1}{b^2+1}+\frac{1}{c^2+1}\ge3-\frac{a+b+c}{2}\)

Cần chứng minh: \(3-\frac{a+b+c}{2}\ge\frac{3}{2}\Leftrightarrow a+b+c\le3\)

Hình như có gì đó sai sai @@

Khách vãng lai đã xóa
zZz Cool Kid_new zZz
15 tháng 9 2020 lúc 17:41

Lời giải kia sai rồi :V Làm cách khác:

Ta có:\(\frac{1}{a^2+1}=\frac{a^2+1-a^2}{a^2+1}=1-\frac{a^2}{a^2+1}\)

Tương tự rồi ta được:

\(LHS=3-\left(\frac{a^2}{a^2+1}+\frac{b^2}{b^2+1}+\frac{c^2}{c^2+1}\right)\)

Bất đẳng thức cần chứng minh tương đương với: 

\(\frac{a^2}{a^2+1}+\frac{b^2}{b^2+1}+\frac{c^2}{c^2+1}\le\frac{3}{2}\)

\(\Leftrightarrow\frac{a^2}{3a^2+3}+\frac{b^2}{3b^2+3}+\frac{c^2}{3c^2+3}\le\frac{1}{2}\)

Ta dễ có được:

\(\frac{4a^2}{3a^2+3}=\frac{4a^2}{3a^2+ab+bc+ca}=\frac{\left(a+a\right)^2}{a\left(a+b+c\right)+2a^2+bc}\le\frac{a^2}{a\left(a+b+c\right)}+\frac{a^2}{2a^2+bc}\)

Tương tự:

\(\frac{4b^2}{3b^2+3}\le\frac{b^2}{b\left(a+b+c\right)}+\frac{b^2}{2b^2+ca};\frac{4c^2}{3c^2+3}\le\frac{c^2}{c\left(a+b+c\right)}+\frac{c^2}{2c^2+ab}\)

\(\Rightarrow LHS\le\frac{1}{4}\left(\frac{a}{a+b+c}+\frac{b}{a+b+c}+\frac{c}{a+b+c}+\Sigma\frac{a^2}{2a^2+bc}\right)=\frac{1}{4}\left(1+\Sigma\frac{a^2}{2a^2+bc}\right)\)

Một cách khác ta dễ có được: \(\Sigma\frac{a^2}{2a^2+bc}\le1\)

Done !

Khách vãng lai đã xóa
Linh Châu
Xem chi tiết
Nguyễn Việt Lâm
3 tháng 7 2020 lúc 12:21

1.

Áp dụng BĐT \(x^2+y^2+z^2\ge xy+yz+zx\)

\(\Rightarrow\left(\sqrt{ab}\right)^2+\left(\sqrt{bc}\right)^2+\left(\sqrt{ca}\right)^2\ge\sqrt{ab}.\sqrt{bc}+\sqrt{ab}.\sqrt{ac}+\sqrt{bc}.\sqrt{ac}\)

\(\Rightarrow ab+bc+ca\ge\sqrt{abc}\left(\sqrt{a}+\sqrt{b}+\sqrt{c}\right)\)

2.

\(\frac{ab}{c}+\frac{bc}{a}\ge2\sqrt[]{\frac{ab.bc}{ca}}=2b\) ; \(\frac{ab}{c}+\frac{ac}{b}\ge2a\) ; \(\frac{bc}{a}+\frac{ac}{b}\ge2c\)

Cộng vế với vế:

\(2\left(\frac{ab}{c}+\frac{bc}{a}+\frac{ac}{b}\right)\ge2\left(a+b+c\right)\)

\(\Leftrightarrow\frac{ab}{c}+\frac{bc}{a}+\frac{ac}{b}\ge a+b+c\)

3.

Từ câu b, thay \(c=1\) ta được:

\(ab+\frac{b}{a}+\frac{a}{b}\ge a+b+1\)

Nguyễn Việt Lâm
3 tháng 7 2020 lúc 12:25

4.

\(\frac{a^3}{b}+\frac{b^3}{c}+\frac{c^3}{a}=\frac{a^4}{ab}+\frac{b^4}{bc}+\frac{c^4}{ac}\ge\frac{\left(a^2+b^2+c^2\right)}{ab+bc+ca}\)

\(\Rightarrow\frac{a^3}{b}+\frac{b^3}{c}+\frac{c^3}{a}\ge\frac{\left(ab+bc+ca\right)^2}{ab+bc+ca}=ab+bc+ca\)

Dấu "=" xảy ra khi \(a=b=c\)

5.

\(\frac{a}{bc}+\frac{b}{ca}\ge2\sqrt{\frac{ab}{bc.ca}}=\frac{2}{c}\) ; \(\frac{a}{bc}+\frac{c}{ab}\ge\frac{2}{b}\) ; \(\frac{b}{ca}+\frac{c}{ab}\ge\frac{2}{a}\)

Cộng vế với vế:

\(2\left(\frac{a}{bc}+\frac{b}{ca}+\frac{c}{ab}\right)\ge2\left(\frac{1}{a}+\frac{1}{b}+\frac{1}{c}\right)\)

\(\Rightarrow\frac{a}{bc}+\frac{b}{ca}+\frac{c}{ab}\ge\frac{1}{a}+\frac{1}{b}+\frac{1}{c}\)

l҉o҉n҉g҉ d҉z҉
8 tháng 1 2021 lúc 22:31

1. bđt được viết lại thành

\(ab+bc+ca\ge a\sqrt{bc}+b\sqrt{ac}+c\sqrt{ab}\)

Theo bđt AM-GM thì :

\(ab+bc\ge2\sqrt{ab\cdot bc}=2\sqrt{ab^2c}=2b\sqrt{ac}\)

Tương tự : \(bc+ca\ge2c\sqrt{ab}\)\(ab+ca\ge2a\sqrt{bc}\)

Cộng vế với vế

=> \(2\left(ab+bc+ca\right)\ge2\left(a\sqrt{bc}+b\sqrt{ac}+c\sqrt{ab}\right)\)

=> \(ab+bc+ca\ge a\sqrt{bc}+b\sqrt{ac}+c\sqrt{ab}\)( đpcm )

Dấu "=" xảy ra <=> a=b=c

Khách vãng lai đã xóa
Nobita Kun
Xem chi tiết
Nguyễn Tất Đạt
14 tháng 10 2018 lúc 22:25

Dễ thấy: \(a^2+b^2+c^2\ge ab+bc+ca\). Mà \(a^2+b^2+c^2\le3\) nên \(ab+bc+ca\le3\)

Áp dụng BĐT Schwarz cho 2 bộ số: (1;1;1) và (1+ab;1+bc;1+ca) ta có:

\(\frac{1}{1+ab}+\frac{1}{1+bc}+\frac{1}{1+ca}\ge\frac{9}{3+ab+bc+ca}\ge\frac{9}{3+3}=\frac{3}{2}\)(Do \(ab+bc+ca\le3\))

=> ĐPCM. Dấu "=" xảy ra <=> a=b=c=1.

abc081102
Xem chi tiết
Luân Đào
Xem chi tiết
tthnew
28 tháng 7 2019 lúc 19:15

Đề chơi căng nhỉ?

a) Dễ chứng minh VP =< 3

BĐT \(\Leftrightarrow\left(\frac{a+b}{1+a}-1\right)+\left(\frac{b+c}{1+b}-1\right)+\left(\frac{c+a}{1+c}-1\right)\ge0\)

\(\Leftrightarrow\frac{b-1}{1+a}+\frac{c-1}{1+b}+\frac{a-1}{1+c}\ge0\)

\(\Leftrightarrow\frac{\left(b-1\right)^2}{\left(1+a\right)\left(b-1\right)}+\frac{\left(c-1\right)^2}{\left(1+b\right)\left(c-1\right)}+\frac{\left(a-1\right)^2}{\left(1+c\right)\left(a-1\right)}\) >=0

Áp dụng BĐT Cauchy-Schwarz dạng Engel vào VT ta có đpcm.

P/s: Èo, sao đơn giản thế nhỉ? Em có làm sai chỗ nào chăng?

Trần Phúc Khang
28 tháng 7 2019 lúc 21:56

a, Ta có \(\frac{a+b}{a+1}=\frac{\left(a+b\right)\left(a+1\right)-a\left(a+b\right)}{a+1}=a+b-\frac{a\left(a+b\right)}{a+1}\)

\(\frac{1}{a+1}\le\frac{a+1}{4a}\)

=> \(\frac{a+b}{1+a}\ge a+b-\frac{\left(a+1\right)\left(a+b\right)}{4}=\frac{3}{4}\left(a+b+c\right)-\frac{1}{4}a^2-\frac{1}{4}ab\)

Khi đó

\(Vt\ge\frac{3}{2}\left(a+b+c\right)-\frac{1}{4}\left(a^2+b^2+c^2\right)-\frac{1}{4}\left(ab+bc+ac\right)\)

=> \(VT\ge\frac{9}{2}-\frac{1}{4}\left(9-2ab-2bc-2ac\right)-\frac{1}{4}\left(ab+bc+ac\right)\)

=> \(VT\ge\frac{9}{4}+\frac{1}{4}\left(ab+bc+ac\right)\)

Lại có \(ab+bc+ac\le\frac{1}{3}\left(a+b+c\right)^2=3\)

=> \(VT\ge ab+bc+ac\)(ĐPCM)

Dấu bằng xảy ra khi a=b=c=1

b,Ta có \(\frac{a}{b\left(a+b^2\right)}=\frac{a+b^2-b^2}{b\left(a+b^2\right)}=\frac{1}{b}-\frac{b}{a+b^2}\)

\(a+b^2\ge2b\sqrt{a}\)

=> \(\frac{a}{b\left(a+b^2\right)}\ge\frac{1}{b}-\frac{1}{2\sqrt{a}}\)

Lại có \(\frac{1}{\sqrt{a.1}}\le\frac{1}{2}\left(\frac{1}{a}+1\right)\)

=> \(\frac{a}{b\left(a+b^2\right)}\ge\frac{1}{b}-\frac{1}{4}.\left(\frac{1}{a}+1\right)\)

Khi đó

\(VT\ge\frac{3}{4}\left(\frac{1}{a}+\frac{1}{b}+\frac{1}{c}\right)-\frac{3}{4}\)

\(\frac{1}{a}+\frac{1}{b}+\frac{1}{c}\ge\frac{9}{a+b+c}=3\)

=> \(VT\ge\frac{9}{4}-\frac{3}{4}=\frac{3}{2}\)(ĐPCM)

Dấu bằng xảy ra khi a=b=c=1

Nguyễn Quang Định
29 tháng 7 2019 lúc 10:24

Bất đẳng thức được viết lại thành

\(\sum\frac{3-a}{1+a}\ge ab+bc+ca\)

\(ab+bc+ca\le3\) nên ta chỉ cần chứng minh

\(\sum\frac{3-a}{1+a}\ge3\)

Ta chứng minh bất đẳng thức phụ sau

\(\frac{3-a}{1+a}\ge2-a\)

\(\Leftrightarrow\left(a-1\right)^2\ge0\)

Thiết lập các bất đẳng thức tương tự ta có điều phải chứng minh

asssssssaasawdd
Xem chi tiết
Hà Lê
Xem chi tiết
Thắng Nguyễn
9 tháng 7 2017 lúc 17:24

Lần sau đăng ít 1 thôi đăng nhiều ngại làm, bn đăng nhiều nên tui hướng dẫn sơ qua thôi tự làm đầy đủ vào vở

Bài 1:

Áp dụng BĐT AM-GM ta có:

\(a^4+b^4\ge2a^2b^2;b^4+c^4\ge2b^2c^2;c^4+a^4\ge2c^2a^2\)

Cộng theo vế 3 BĐT trên rồi thu gọn

\(a^4+b^4+c^4\ge a^2b^2+b^2c^2+c^2a^2\)

Áp dụng tiếp BĐT AM-GM

\(a^2b^2+b^2c^2=b^2\left(a^2+c^2\right)\ge2b^2ac\)

Tương tự rồi cộng theo vế có ĐPCM

Bài 2:

Quy đồng  BĐT trên ta có:

\(\frac{a^2}{b^2}+\frac{b^2}{a^2}-\frac{a}{b}-\frac{b}{a}\ge0\)

\(\Leftrightarrow\frac{\left(a-b\right)^2\left(a^2+ab+b^2\right)}{a^2b^2}\ge0\) (luôn đúng)

Bài 4: Áp dụng BĐT AM-GM 

\(a^3+b^3=\left(a+b\right)\left(a^2-ab+b^2\right)\)

\(\ge\left(a+b\right)\left(2ab-ab\right)=ab\left(a+b\right)\)

\(\Rightarrow\frac{a^3+b^3}{ab}\ge\frac{ab\left(a+b\right)}{ab}=a+b\)

Tương tự rồi cộng theo vế

Bài 5: sai đề tự nhien có dấu - :v nghĩ là +

Game Master VN
9 tháng 7 2017 lúc 9:54

ai k mình k lại [ chỉ 3 người đầu tiên mà trên 10 điểm hỏi đáp ]

 
Ninh Nguyễn Trọng
Xem chi tiết
zZz Cool Kid_new zZz
8 tháng 1 2020 lúc 18:19

\(\frac{1}{a}+\frac{1}{b}+\frac{1}{c}\ge\frac{9}{a+b+c}\ge\frac{9}{6}=\frac{3}{2}\)

\(\frac{1}{ab}+\frac{1}{ac}+\frac{1}{bc}\ge\frac{9}{ab+bc+ca}\ge\frac{27}{\left(a+b+c\right)^2}=\frac{27}{36}=\frac{3}{4}\)

\(\frac{1}{abc}\ge\frac{1}{\left(\frac{a+b+c}{3}\right)^3}=\frac{27}{\left(a+b+c\right)^3}\ge\frac{27}{6^3}=\frac{1}{8}\)

Cộng lại ta được:

\(1+\frac{1}{a}+\frac{1}{b}+\frac{1}{c}+\frac{1}{ab}+\frac{1}{bc}+\frac{1}{ca}+\frac{1}{abc}\ge\frac{27}{8}\left(đpcm\right)\)

Dấu "=" xảy ra tại \(a=b=c=2\)

Khách vãng lai đã xóa